Does the conclusion escape you? Has understanding the tone of the passage gotten you down? Get help here.
akhp77
Students
 
Posts: 114
Joined: Wed Mar 24, 2010 7:25 pm
 

DARPA

by akhp77 Sat May 15, 2010 9:34 am

Source: Manhattan GMAT CR Guide 4th Ed, Page No 144, Chapter 7
-----------------------------------------

Supporters of a costly new defense advanced research projects Agency (DARPA) initiative assert that the project will benefit industrial companies as well as military itself. In many instances, military research has resulted in technologies that have fueled corporate development and growth, and this pattern can be expected to continue.

Each of the following, if true, serves to weaken the argument above EXCEPT:

A: The research initiative will occupy many talented scientists, many of whom would otherwise have worked for private corporations.
B: In past decades, DARPA has adopted an increasingly restrictive stance regarding the use of intellectual property resulting from its research.
C: A high proportion of the government resources directed towards the initiative would ordinarily have gone to tax subsidies for various business.
D: The research initiative is focused on materials development through nano-technology, which is thought to have many commercial applications.
E: The DARPA research makes use of manufacturing processes the would be cost-prohibitive for most companies to replicate.

OA: D

I understood stimulus but Logically, I could not understood any of the choices that how they strengthened or weakened.

Can you please help me on this CR?
tim
Course Students
 
Posts: 5665
Joined: Tue Sep 11, 2007 9:08 am
Location: Southwest Airlines, seat 21C
 

Re: DARPA

by tim Tue Jun 01, 2010 12:48 pm

i'm guessing any instructor who tried to answer your question would likely end up repeating what's already in the guide or something very similar. If you want some further explanation, it would help if you could tell us what didn't make sense about the specific explanations the CR guide provided for how each answer choice strengthened or weakened the conclusion..
Tim Sanders
Manhattan GMAT Instructor

Follow this link for some important tips to get the most out of your forum experience:
https://www.manhattanprep.com/gmat/forums/a-few-tips-t31405.html
JbhB682
Course Students
 
Posts: 520
Joined: Fri May 16, 2014 2:13 pm
 

Re: DARPA

by JbhB682 Fri Oct 02, 2020 12:24 pm

Hi, quick question on C

Per the OA solution C is a weakener

However, is C a weaken-er in every possible scenario ?

Per C, Tax subsidies would have been given to these businesses instead of the DARPA initiative.

Now given, businesses are no longer getting these tax subsidies (given its going towards this DARPA initiative instead) , this is considered a reason why the DARPA initiative would not HELP businesses.

But it's quite possible that

-- the R&D developed as a result of the DARPA initiative and given to businesses could BE MORE PROFITABLE in the long run compared to the tax subsidies.

I thought about this side scenario when I read C and I was not sure if C was a weakener in ALL possible scenario with regards to tax subsidies as the DARPA initiative could be more profitable for the companies comparably to the tax subsidies
Sage Pearce-Higgins
Forum Guests
 
Posts: 1336
Joined: Thu Apr 03, 2014 4:04 am
 

Re: DARPA

by Sage Pearce-Higgins Mon Oct 05, 2020 4:46 am

In Strengthen / Weaken problems there's a level of "reasonable vagueness" that we need to get used to. When dealing with words (rather that numbers) we have to acknowledge that there's going to be some imprecision, and that it's rare for things to be 100% precise. I agree that answer choice C doesn't weaken the conclusion (that DARPA will 'benefit industrial companies') in every possible scenario. As you rightly state, we'd need a lot more information about the types of investment, the long-term outcomes, etc. to make that call. However, answer C gives some reasonable grounds for weakening the conclusion, unlike answer D, which strengthens the conclusion.